College Math
Topics in undergraduate and graduate studies
Topics in undergraduate and graduate studies
3
M
G
BBookmark
VNew Topic
kLocked
College Math
Topics in undergraduate and graduate studies
Topics in undergraduate and graduate studies
3
M
G
BBookmark
VNew Topic
kLocked
No tags match your search
Msuperior algebra solved
calculus
real analysis
linear algebra
superior algebra
complex analysis
advanced fields
probability and stats
number theory
topology
Putnam
college contests
articles
function
integration
calculus computations
real analysis unsolved
limit
algebra
trigonometry
matrix
logarithms
derivative
superior algebra unsolved
polynomial
abstract algebra
geometry
inequalities
vector
group theory
linear algebra unsolved
probability
advanced fields unsolved
analytic geometry
domain
induction
LaTeX
Ring Theory
3D geometry
complex analysis unsolved
complex numbers
Functional Analysis
geometric transformation
superior algebra solved
real analysis theorems
search
parameterization
quadratics
real analysis solved
limits
ratio
No tags match your search
MG
Topic
First Poster
Last Poster
IMC 1994 D1 P5
j___d 5
N
Today at 5:39 PM
by krigger
a) Let
,
and let
be periodic with period
. Prove that
has a limit as
and

b) Find
![$f\in C[0,b]$](http://latex.artofproblemsolving.com/8/9/8/898aec0300888e562794b7a551be34f5c2676236.png)






b) Find

5 replies
2023 Putnam A2
giginori 21
N
Today at 3:32 PM
by pie854
Let
be an even positive integer. Let
be a monic, real polynomial of degree
; that is to say,
for some real coefficients
. Suppose that
for all integers
such that
. Find all other real numbers
for which
.











21 replies
Putnam 2019 A1
awesomemathlete 33
N
Today at 3:25 PM
by cursed_tangent1434
Source: 2019 William Lowell Putnam Competition
Determine all possible values of
where
,
, and
are nonnegative integers.




33 replies
IMC 1994 D1 P2
j___d 5
N
Today at 3:11 PM
by krigger
Let
,
,
and
for
. Prove that
and give an example where
.







5 replies
A Construction in Multivariable Analysis
MrOrange 0
Today at 2:11 PM
Source: Garling's A COURSE IN MATHEMATICAL ANALYSIS
Construct a continuous real valued function
on
for which
and for which


![\[
\lim_{R \to \infty} \int_{\|x\|_2 \leq R} f(x) \, dx = 0
\]](http://latex.artofproblemsolving.com/4/6/9/469f69ee93ae8d1de73c31ceb066253751aa9775.png)
![\[
\lim_{R \to \infty} \int_{\|x\|_\infty \leq R} f(x) \, dx \text{ does not exist.}
\]](http://latex.artofproblemsolving.com/d/f/a/dfadbdd902375b9f67cd2dd0dbb494978bd9e75c.png)
0 replies
Possible values of determinant of 0-1 matrices
mathematics2004 4
N
Today at 1:56 PM
by loup blanc
Source: 2021 Simon Marais, A3
Let
be the set of all
matrices with at most two entries in each row equal to
and all other entries equal to
.
Determine the size of the set
.
Here
denotes the determinant of the matrix
.




Determine the size of the set

Here


4 replies
ISI UGB 2025
Entrepreneur 1
N
Today at 1:49 PM
by Knight2E4
Source: ISI UGB 2025
1.)
Suppose
is differentiable and
Show that for some 
3.)
Suppose
is differentiable with
If
then show that 
4.)
Let
be the unit circle in the complex plane. Let
be the map given by
We define
and
for
The smallest positive integer
such that
is called period of
Determine the total number of points
of period 
6.)
Let
denote the set of natural numbers, and let
be nine distinct tuples in
Show that there are
distinct elements in the set
whose product is a perfect cube.
8.)
Let
and let
be positive integers such that
Prove that
and determine when equality holds.
Suppose



3.)
Suppose
![$f:[0,1]\to\mathbb R$](http://latex.artofproblemsolving.com/e/a/0/ea02b990406104357c11912a8bf57bea7eeb519d.png)

![$|f'(x)|\le f(x)\;\forall\;x\in[0,1],$](http://latex.artofproblemsolving.com/b/9/d/b9d6088657cd85b37190b98abac1d94a18bb5916.png)

4.)
Let











6.)
Let





8.)
Let




1 reply
Recurrence trouble
SomeonecoolLovesMaths 3
N
Today at 1:44 PM
by Knight2E4
Let
be real numbers. Define
and
.
Prove that
and hence find the limit.



Prove that

3 replies
Trigo or Complex no.?
hzbrl 5
N
Today at 9:20 AM
by GreenKeeper
(a) Let
, where
. Verify by direct substitution that
satisfies the quadratic equation
and deduce that the value of
is
.
(b) Let
. Show that 
(c) If
, show that the value of
is
.
I could solve (a) and (b). Can anyone help me with the 3rd part please?






(b) Let


(c) If



I could solve (a) and (b). Can anyone help me with the 3rd part please?
5 replies
2023 Putnam A1
giginori 29
N
Yesterday at 10:52 PM
by kidsbian
For a positive integer
, let
. Find the smallest
such that
.




29 replies
